what digit is in the

Answers

Answer 1

Let:

Mp = Marked price = $310

r = Rate of discount = 20% = 0.2

D = Discount

Sp = Sale price

The discount will be given by:

[tex]\begin{gathered} D=r\cdot Mp \\ D=0.2\cdot310 \\ D=62 \end{gathered}[/tex]

And the sale price will be:

[tex]\begin{gathered} Sp=Mp-D \\ Sp=310-62 \\ Sp=248 \end{gathered}[/tex]


Related Questions

What is the y-intercept of the line x+2y=-14? (0,7) (-7,0) (0,-7) (2,14)

Answers

[tex]\begin{gathered} \text{First, we need to isolate y} \\ 2y=-x-14 \\ y=\frac{-x-14}{2} \\ y=\frac{-x}{2}-\frac{14}{2} \\ y=-\frac{x}{2}-7 \\ -7\text{ represents the y-intercept} \\ \text{When you write as a point it would be (0, -7)} \end{gathered}[/tex]

write each of the following numbers as a power of the number 2

Answers

Answer

The power on 2 is either -3.5 in decimal form or (-7/2) in fraction form.

Explanation

To do this, we have to first note that

[tex]\begin{gathered} \sqrt[]{2}=2^{\frac{1}{2}} \\ \text{And} \\ 16=2^4 \end{gathered}[/tex]

So, we can then simplify the given expression

[tex]\begin{gathered} \frac{\sqrt[]{2}}{16}=\frac{2^{\frac{1}{2}}}{2^4}=2^{\frac{1}{2}-4} \\ =2^{0.5-4} \\ =2^{-3.5} \\ OR \\ =2^{\frac{-7}{2}} \end{gathered}[/tex]

Hope this Helps!!!

Factor Problem Completely 16n^3 - 56n^2 + 8n - 28

Answers

Given

The equation is given as

[tex]16n^3-56n^2+8n-28[/tex]

Explanation

Factorisation the equation,

[tex]4(4n^3-14n^2+2n-7)[/tex]

Factorise the polynomial.

[tex]4(2n-7)(2n^2+1)[/tex]Answer

Hence the answer is

[tex]4(2n-7)(2n^2+1)[/tex]

1. P(video games and kid is 10 to 12 years old)2. P(basketball/kid is 13 to 15 years old)3. P(kid is 13 to 15 years old/basketball)4. P(darts/kid is 10 to 15 years old)5. P(basketball and darts)6. P(basketball and kid is 13 to 18 years old)Answer the following problems about two way frequency tables make sure to reduce your fraction.

Answers

1. P(video games and kid is 10 to 12 years old)

[tex]\begin{gathered} P(video\text{ games and kid i 10 to 12 years old)} \\ =\text{ }\frac{number\text{ of kids 10 - 12 years old playing video games}}{total\text{ number of students}} \\ =\text{ }\frac{17}{143} \end{gathered}[/tex]

Therefore,

The P(video games and kid is 10 to 12 years old) = 17/143

2. P(basketball/kid is 13 to 15 years old)

[tex]\begin{gathered} P\mleft(basketball/kid\text{ is 13 to 15 years old}\mright)\text{ } \\ =\text{ }\frac{number\text{ of kids 13 - 15 years old playing basketball}}{number\text{ of kids of age 13 to 15 years old}} \\ =\text{ }\frac{14}{45} \end{gathered}[/tex]

P(basketball/kid is 13 to 15 years old) = 14/45

3. P(kid is 13 to 15 years old/basketball)

[tex]\begin{gathered} P(\text{kid is 13 to 15 years old / basket ball)} \\ =\text{ }\frac{number\text{ of kids aged 13 to 15 years old }}{number\text{ of kids playing basketball}} \\ =\text{ }\frac{14}{54} \\ =\text{ }\frac{7}{27} \end{gathered}[/tex]

P(kid is 13 to 15 years old/basketball) = 7/27

4. P(darts/kid is 10 to 15 years old)

[tex]\begin{gathered} P(\text{darts / kid is 10 to 15 years old)} \\ =\text{ }\frac{number\text{ of kids age 10 to 15 playing darts}}{\text{number of kids age 10 to 15}} \\ =\text{ }\frac{kids\text{ age 10 to 12 + age 13 to 15 playing darts}}{\text{kids age 10 to 12 + age 13 to 15}} \\ =\text{ }\frac{12\text{ + 15}}{34\text{ + 45}} \\ =\text{ }\frac{27}{79} \end{gathered}[/tex]

P(darts/kid is 10 to 15 years old) = 27/79

5. P(basketball and darts)

[tex]\begin{gathered} P(basketball\text{ and darts)} \\ \sin ce\text{ there are no kids playing basketball and darts at the } \\ \text{same time} \\ \text{then,} \\ P(basketball\text{ and darts) = 0} \end{gathered}[/tex]

P(basketball and darts) = 0

6. P(basketball and kid is 13 to 18 years old)

[tex]\begin{gathered} P(\text{basketball and kid is 13 to 18 years old)} \\ =\text{ }\frac{number\text{ of kids 13 to 18 years playing basket}}{nu\text{mber of kid 13 to 18 years }} \\ =\text{ }\frac{\text{kids 13 to 15 years + 16 - 18 years playing basketball}}{\text{kids 13 to 15years + 16 to 18 years}} \\ =\text{ }\frac{14\text{ + 18}}{45\text{ + 35}} \\ =\text{ }\frac{32}{80} \\ =\frac{2}{5} \end{gathered}[/tex]

P(basketball and kid is 13 to 18 years old) = 2/5

Please assist me. I have no idea how to start this equation

Answers

Part a

Remember that the linear equation in slope-intercept form is

y=mx+b

where

m is the slope or unit rate

b is the y-intercept or initial value

In this problem

the equation is of the form

C=m(n)+b

where

m=8.50

b=350

therefore

C=8.50n+350

Part b

A reasonable domain for n (number of cups)

Remember that the number of cups cannot be a negative number

so

the domain is the interval [0, infinite)

but a reasonable domain could be [0, 500]

Find out the range

For n=0 -----> C=350

For n=500 ----> C=8.50(500)+350=2,100 ZAR

the range is the interval [350,2,100]

Part c

calculate the cost

For n=100 cups ----> C=8.50(100)+350=1,200 ZAR

For n=200 cups ----> C=8.50(200)+350=2,050 ZAR

For n=400 cups ---> C=8.50(400)+350=3,750 ZAR

Part d

Average cost

Divide the total cost by the number of cups

For 100 cups ------> 1,200/100=12 ZAR per cup

For 200 cups ----> 2,050/200=10.25 ZAR per cup

For 400 cups ----> 3,750/400=9.38 ZAR per cup

Part e

it is better to order more cups, to reduce the initial ZAR 350 cost.

Part f

In this problem we have the ordered pairs

(200, 2150) and (400, 3750)

Find out the slope m

m=(3750-2150)/(400-200)

m=8 ZAR per cup

Find out the linear equation

C=mn+b

we have

m=8

point (200,2150)

substitute and solve for b

2150=8(200)+b

b=2150-1600

b=550

therefore

The linear equation is

C=8n+550

Part g

A reasonable domain could be [0, 600]

Find out the range

For n=0 ------> C=550

For n=600 ----> C=8(600)+550=5,350

The range is the interval [550,5350]

Part h

The gradient is the same as the slope

so

slope=8

that means ----> the cost of each cup is 8 ZAR

Part i

For n=600

C=8(600)+550=5,350 ZAR

Part j

we have the inequality

8n+550 < 8.50n+350

Solve for x

550-350 < 8.50n-8n

200 < 0.50n

400 < n

Rewrite

n > 400

For orders more than 400 cups is more effective to order from Cupomatic

Verify

For n=401

C=8n+550=8(401)+550=3,758 ZAR

C=8.50n+350=8.5(401)+350=3,758.5 ZAR

the cost is less in CUPOMATIC, is ok

the answer is

For orders more than 400 cups is more effective to order from Cupomatic

Can you please help me find the area of the shaded triangle? Thank you :)

Answers

Area of shaded triangle = Area of triangle - area of circle

Area of triangle = 1/2 x base x height

Base= 16 yds

Height= 19 yds

Area of triangle = 1/2 x 16 x 19 = 8 x 19 =152 square yard

[tex]\begin{gathered} \text{Area of circle = }\pi\times r^2 \\ \pi=3.14 \\ r=5\text{yds} \\ \text{Area of circle = 3.14 }\times5^2=78.5yard^2 \end{gathered}[/tex]

Area of shaded triangle = 152 - 78.5 =73.5 square yard

hello can you help me with this trigonometry question and this a homework assignment

Answers

You have:

sin 2A = -√7/4

In order to determine the value of sin A, first calculate the value of angle A by using sin⁻¹ over the previous equation, just as follow:

sin⁻¹(sin 2A) = sin⁻¹(-√7/4) In this way you cancel out the sin

2A = -41.41° divide by 2 both sides

A = -41.41°/2

A = -20.705°

however, take into account that angle A is in the third quadrant. Then, it is necessary to consider the result A=-20.705° is respect to the negative x-axis.

To obtain the angle respect the positive x-axis (the normal way), you simply sum 180° to 20.705°:

20.705 + 180° = 200.705°

Next, use calculator to calculate sinA:

sin(200.705°) = -0.3535

Hi, can you help me to solve thisexercise, please!!For cach polynomial, LIST all POSSIBLE RATIONAL ROOTS•Find all factors of the leading coefficient andconstant value of polynonnal.•ANY RATIONAL ROOTS =‡ (Constant Factor over Leading Coefficient Factor)6x^3+7x^2-3x-1

Answers

[tex]\begin{gathered} Possible\: Roots\colon\pm1,\pm\frac{1}{2},\pm\frac{1}{3},\pm\frac{1}{6} \\ Actual\: Rational\: Roots\colon\: None \end{gathered}[/tex]

1) We can do this by listing all the factors of -1, and the leading coefficient 6. So, we can write them as a ratio this way:

[tex]\frac{p}{q}=\pm\frac{1}{1,\:2,\:3,\:6}[/tex]

Note that p stands for the constant and q the factors of that leading coefficient

2) Now, let's test them by plugging them into the polynomial. If it is a rational root it must yield zero:

[tex]\begin{gathered} 6x^3+7x^2-3x+1=0 \\ 6(\pm1)^3+7(\pm1)^2-3(\pm1)+1=0 \\ 71\ne0,5\ne0 \\ \frac{1}{2},-\frac{1}{2} \\ 6(\pm\frac{1}{2})^3+7(\pm\frac{1}{2})^2-3(\pm\frac{1}{2})+1=0 \\ 2\ne0,\frac{7}{2}\ne0 \\ \\ 6(\pm\frac{1}{3})^3+7(\pm\frac{1}{3})^2-3(\pm\frac{1}{3})+1=0 \\ 1\ne0,\frac{23}{9}\ne0 \\ \frac{1}{6},-\frac{1}{6} \\ 6(\frac{1}{6})^3+7(\frac{1}{6})^2-3(\frac{1}{6})+1=0 \\ \frac{13}{18}\ne0,-\frac{5}{3}\ne0 \end{gathered}[/tex]

3) So the possible roots are:

[tex]\pm1,\pm\frac{1}{2},\pm\frac{1}{3},\pm\frac{1}{6}[/tex]

But there are no actual rational roots.

Jonathan is playing a game or a regular board that measures 60 centimeters long and 450 mm wide. which measurement is closest to the perimeter of the Jonathan's game board in meters?

Answers

According to the problem, the length is 60 cm and the width is 450 mm.

Let's transform 450mm to cm. We know that 1 cm is equivalent to 10 mm. So,

[tex]450\operatorname{mm}\times\frac{1\operatorname{cm}}{10\operatorname{mm}}=45\operatorname{cm}[/tex]

Then, we use the perimeter formula for rectangles.

[tex]P=2(w+l)[/tex]

Where w = 45 cm and l = 60 cm.

[tex]\begin{gathered} P=2(45\operatorname{cm}+60\operatorname{cm})=2(105cm) \\ P=210\operatorname{cm} \end{gathered}[/tex]The perimeter is 210 centimeters long.

However, we know that 1 meter is equivalent to 100 centimeters.

[tex]P=210\operatorname{cm}\cdot\frac{1m}{100\operatorname{cm}}=2.1m[/tex]

Hence, the perimeter, in meters, is 2.1 meters long.

Option A is the answer.

What is the standard form of the equation of a line passing through points (2,3) and (2,-5)?

Answers

Answer:

[tex]x\text{ = 2}[/tex]

Explanation:

Here, we want to find the standard form of the equation

We have the standard form as:

[tex]Ax\text{ + By = C}[/tex]

We can arrive at this using the two-points form:

This is:

[tex]\frac{y_2-y_1}{x_2-x_1}\text{ = }\frac{y-y_1}{x-x_1}[/tex]

(x1,y1) = (2,3)

(x2,y2) = (2,-5)

Now, as we can see, the line is a vertical line since the x-value is the same

Thus, we have it that:

[tex]x\text{ = c}[/tex]

where c will represent the x-intercept

Thus, we have the equation of the line as:

[tex]x\text{ = 2}[/tex]

Describe it and decide if normal curve could be used as model

Answers

Answer:

The symmetric is symmetric

The distribution is unimodal

The mean, median, and mode are equal

A normal distribution is appropriate

Explanation:

The normal distribution is symmetric and unimodal, where the mode, the median, and the mean are equal. This distribution has the following shape

Therefore, the normal curve can be used as a model for the distribution.

So, the answers are:

The symmetric is symmetric

The distribution is unimodal

The mean, median, and mode are equal

A normal distribution is appropriate

Write a multiplication expression to represent each situation. Then find each product and explain its meaning. Ethan burns 650 calories when he runs for 1 hour. Suppose he runs 5 hours in one week.

Answers

We know that

• Ethan burns 650 calories per hour.

If he runs 5 hours we just have to multiply this time with the given rate.

[tex]650\cdot5=3,250[/tex]Therefore, Ethan burns 3,250 calories in 5 hours.

exponent hw.simplify

Answers

Answer:

4

Explanation:

Given the below;

[tex]4m^0[/tex]

To simplify the above, we have to note that any number or variable raised to the power of 0 is 1.

So, we'll have;

[tex]4m^0=4\times1=4[/tex]

StatusRecovery8Help ResourcessAABC ~ AXYZFind the missing side length, s.B.3 65А&Х-ZCross multiplySE ][?] = [ ]153s

Answers

Since triangles ABC and XYZ are similar, the ratio between their corresponding sides is constant; thus,

[tex]\begin{gathered} \frac{AB}{XY}=\frac{BC}{YZ} \\ \Rightarrow\frac{3}{5}=\frac{6}{s} \end{gathered}[/tex]

Solving for s,

[tex]\begin{gathered} \frac{3}{5}=\frac{6}{s} \\ \Rightarrow\frac{3}{5}\cdot s=\frac{6}{s}\cdot s \\ \Rightarrow\frac{3s}{5}=6 \\ \Rightarrow\frac{3s}{5}\cdot5=6\cdot5 \\ \Rightarrow3s=30 \\ \Rightarrow s=\frac{30}{3} \\ \Rightarrow s=10 \end{gathered}[/tex]

Thus, the result of the cross multiplication is 3s=30 and the answer is s=10

The pentagonal prism below has a height of 13 units and a volume of 247 units ^3. Find the area of one of its bases.

Answers

• Volume of pentagonal prism = area of base x height

Volume = 247 unis^3

height = 13 units

Replacing:

V = A x h

A = V / h

A = 247/13 = 19 units^2

Consider the expression 6+(x+3)^2. Tabulate at least SIX different values of the expression.​

Answers

Considering the expression 6+(x+3)^2. the table of at least SIX different values of the expression is

x               y

0            15

1             22

2            31

3            42

4            55

5            70

How to determine the he table of at least SIX different values of the expression

The table is completed by substituting the values of x in the given expression as follows

6 + (  x + 3 )^2

for x = 0, y = 6 + ( 0 + 3) ^2 = 15

for x = 1, y = 6 + ( 1 + 3) ^2 = 22

for x = 2, y = 6 + ( 2 + 3) ^2 = 31

for x = 3, y = 6 + ( 3 + 3) ^2 = 42

for x = 4, y = 6 + ( 4 + 3) ^2 = 55

for x = 5, y = 6 + ( 5 + 3) ^2 = 70

Learn more about table completion here:

https://brainly.com/question/29232584

#SPJ1

Given that angle A lies in Quadrant III and sin(A)= −17/19, evaluate cos(A).

Answers

As we know;

[tex]sin^2(x)+cos^2(x)=1[/tex]

We will use this equality. We take the square of the sine of the given angle and subtract it from [tex]1[/tex].

[tex]sin^2(A)=(-\frac{17}{19} )^2=\frac{289}{361}[/tex][tex]sin^2(A)+cos^2(A)=1[/tex][tex]sin^2(A)=1-cos^2(A)[/tex][tex]\frac{289}{361}=1-cos^2(A)[/tex][tex]cos^2(A)=1-\frac{289}{361} =\frac{72}{361}[/tex][tex]\sqrt{cos^2(A)} =cos(A)[/tex][tex]\sqrt{\frac{72}{361} }=\frac{6\sqrt{2} }{19}[/tex]

In the third region the sign of cosines is negative. Therefore, our correct answer should be as follows;

[tex]cos(A)=-\frac{6\sqrt{2} }{19}[/tex]

Shanice has 4 times as much many pairs of shoes as does her brother Ron. If Shanice gives Ron 12 pairs of shoes, she will have twice as many pairs of shoes as Ron does. How many pairs of shoes will Shanice have left after she gives Ron the shoes?

Answers

Let's define:

x: pairs of shoes of Shanice

y: pairs of shoes of Ron

Shanice has 4 times as much many pairs of shoes as does her brother Ron, means:

x = 4y (eq. 1)

If Shanice gives Ron 12 pairs of shoes, she will have twice as many pairs of shoes as Ron does, means:

x - 12 = 2y (eq. 2)

Replacing equation 1 into equation 2:

4y - 12 = 2y

4y - 2y = 12

2y = 12

y = 12/2

y = 6

and

x = 4*6 = 24

After she gives Ron the shoes, she will have left 24-12 = 12 pairs of shoes

solve 6 + 5 on the sqr root of 249 - 2x = 7

Answers

ANSWER

x = 124

EXPLANATION

First we have to clear the term that contains x in the equation. In this case, this term is the second term. So we have tu subtract 6 from both sides of the equation:

[tex]\begin{gathered} 6-6+\sqrt[5]{249-2x}=7-6 \\ \sqrt[5]{249-2x}=1 \end{gathered}[/tex]

Then, we have to eliminate the root. Note that in the expression inside the root there are two terms. To do this, we have to apply the "opposite" operation on both sides of the equation, which in this case is exponent 5:

[tex]\begin{gathered} (\sqrt[5]{249-2x})^5=1^5 \\ 249-2x=1 \end{gathered}[/tex]

Now we do something similar to the first step. We want to leave on one side of the equation only the term that contains x and the rest on the other side. To do this we can either add 2x on both sides, or subtract 249 from both sides. We'll apply the first option because then we'll have a positive coefficient for x:

[tex]\begin{gathered} 249-2x+2x=1+2x \\ 249=1+2x \end{gathered}[/tex]

However, we now have to subtract 1 from both sides of the equation:

[tex]\begin{gathered} 249-1=1-1+2x \\ 248=2x \end{gathered}[/tex]

Finally, to find x, we have to divide both sides by 2:

[tex]\begin{gathered} \frac{248}{2}=\frac{2x}{2} \\ 124=x \end{gathered}[/tex]

Hence, the solution to the equation is x = 124.

Solve the quadratic equation x2 − 6x + 13 = 0 using the quadratic formula. What is the solution when expressed in the form a ± bi, where a and b are real numbers?

Answers

The given quadratic equation is:

[tex]x^2-6x+13=0[/tex]

The quadratic formula is given by the equation:

[tex]x=\frac{-b\pm\sqrt[]{b^2-4ac^{}}}{2a}[/tex]

From the given quadratic equation;

[tex]a=1;b=-6\text{ and c=13}[/tex]

Thus, we have:

[tex]x=\frac{-(-6)\pm\sqrt[]{(-6)^2-4(1)(13)}}{2(1)}[/tex]

[tex]\begin{gathered} x=\frac{6\pm\sqrt[]{36-52}}{2} \\ x=\frac{6\pm\sqrt[]{-16}}{2} \\ In\text{ complex form, the }\sqrt[]{-16}=4i \\ \text{Thus, we have:} \\ x=\frac{6\pm4i}{2} \\ x=\frac{6}{2}\pm\frac{4i}{2} \\ x=3\pm2i \end{gathered}[/tex]

Hence, the correct option is Option A

i inserted a picture of the question can you please state whether the answer is A,B, C or D check all that apply

Answers

Solution:

In the given figures, angles of the triangle ABC are corresponding equal to triangle DEF and the sides are proportional to each other.

[tex]\frac{AB}{DE}=\frac{BC}{EF}=\frac{AC}{DF}=\frac{1}{2}[/tex]

Thus, the triangle ABC is similar to triangle DEF.

Therefore, the relationship between both triangles is the proportional side lengths.

Both triangles are not of the same size as their sides are not equal.

Both triangles are also not congruent as they do not satisfy any five conditions of congruence.

Hence, the correct option is A.

i inserted a picture of the questioncan you state whether the answer is A, B, C OR D

Answers

Looking at the triangles, they are both right triangles. They have congruent legs = 12. They have congruent acute angles of 45 degerees. Thus, they are congruent triangles. The answer is True

The table shows a linear relationship between x and y. Drag and drop the options provided into the correct boxes to complete the equation. х 1 0 6 -4 41 у 9 -39 The equation that represents the relationship Is y = -8 -41 ON 9 4 O?

Answers

To calculate the equation first we need to choose two points of the table

P1 (1,1)=(x1,y1)

P2(0,9)=(x2,y2)

then we calculated the slope m

[tex]m=\frac{y_2-y_1}{x_2-x_1}[/tex]

substituting the points we have

[tex]m=\frac{9-1}{0-1}=\frac{8}{-1}=-8[/tex]

then we can calculate the equation

[tex](y-y1)=m(x-x1)[/tex][tex](y-1)=-8(x-1)[/tex]

[tex]y-1=-8x+8[/tex]

[tex]y=-8x+8+1[/tex]

the equation is

[tex]y=-8x+9[/tex]

GRAPH each triangle and CLASSIFY the triangle according to its sides and angles.

Answers

Answer:

[tex]\Delta CAT\text{ is an ISOSCELES triangle}[/tex]

Explanation:

To properly classify the traingle, we need to get the length of the sides

To get the length of the sides, we need to get the distance between each two points using the distance between two points formula

Mathematically,we have the formula as:

[tex]D\text{ = }\sqrt[]{(x_2-x_1)^2+(y_2-y_1)^2}[/tex]

Where (x1,y1) refers to the coordiantes of the first point while (x2,y2) refers to the coordinates of the second point

let us get the coordinates of the individual points as seen from the plot shown

C (1,8)

A (5,10)

T (7,6)

So, let us find the distance between each two points

For AC, we have:

[tex]D\text{ = }\sqrt[]{(5-1)^2+(10-8)^2}\text{ = }\sqrt[]{20}[/tex]

For AT, we have:

[tex]D=\sqrt[]{(7-5)^2+(6-10)^2\text{ }}\text{ = }\sqrt[]{20}[/tex]

Lastly, for CT, we have:

[tex]D\text{ = }\sqrt[]{(7-1)^2+(6-8)^2\text{ }}\text{ = }\sqrt[]{40}[/tex]

From our calculations, we can see that AC = AT

If we have a triangle which has two of its sides equal in length (the angle facing these sides would be same too), we call this an isosceles triangle

So, the class of triangle CAT is isosceles triangle

3. The data in the table gives the number of barbeque sauce bottles (y) that are sold with orders of chicken wings (x) for each hour on a given day at Vonn's Grill. Use technology to write an equation for the line of best fit from the data in the table below. Round all values to two decimal places.

Answers

1) Let's visualize the points

2) To find the equation for the line of best fit we'll need to follow some steps.

2.1 Let's find the mean of the x values and the mean of the Y values

2.2 Now It's time to find the slope, with the summation of the difference between each value and the mean of x times each value minus the mean over the square of the difference of the mean of x and x.

To make it simpler, let's use this table:

The slope then is the summation of the 5th column over the 6th column, we're using the least square method

[tex]m=\frac{939.625}{1270.875}=0.7393\cong0.74[/tex]

The Linear coefficient

[tex]\begin{gathered} b=Y\text{ -m}X \\ b=14.625-0.73(19.875) \\ b=0.11625\cong0.12 \end{gathered}[/tex]

3) Finally the equation of the line that best fit is

[tex]y=0.73x+0.12[/tex]

What is the value of 2(3x − 6) − 5y if x = −2 and y = 6?
−6 −18 −54 −78

Answers

Answer:

-54

Step-by-step explanation:

Finding a value means you will get a number answer. Since they said x

x = -2

fill in -2 in place of the x.

also they said

y = 6

so fill in 6 wherever you see a y.

2(3x - 6) - 5y

fill in -2 for x and 6 for y.

= 2(3•-2 - 6) - 5•6

Work on parentheses first. Multiply before adding or subtracting.

= 2(-6 - 6) - 5•6

= 2(-12) - 5•6

Again, multiply before adding or subtracting.

= -24 - 30

= -54

URGENT!! ILL GIVE
BRAINLIEST! AND 100 POINTS

Answers

According to visual inspection, shape A has been rotated 180° counterclockwise about the origin and then translated 1 unit to the left.

What is meant by transformation?

A point, line, or geometric figure can be transformed in one of four ways, each of which affects the shape and/or location of the object. Pre-Image refers to the object's initial shape, and Image, after transformation, refers to the object's ultimate shape and location.

The four basic transformations exist:

TranslationReflectionRotationDilation

According to visual inspection, shape A has been rotated 180° counterclockwise about the origin and then translated 1 unit to the left.

Therefore, the correct answer is option C) translated 1 unit to the left and then rotated 180° counterclockwise about the origin

The complete question is:

Describe the transformation that maps the pre-image A to the image A.

A) translated 8 units up and then reflected across the y-axis

B) translated 8 units down and then reflected across the y-axis

C) translated 1 unit to left and then rotated 180° counterclockwise about the origin

D) translated 1 unit to right and then rotated 180° counterclockwise about the origin.

To learn more about transformation refer to:

https://brainly.com/question/2689696

#SPJ13

One group (A) contains 155 people. One-fifth of the people in group A will be selected to win $20 fuel cards. There is another group (B) in a nearby town that will receivethe same number of fuel cards, but there are 686 people in that group. What will be the ratio of nonwinners in group A to nonwinners in group B after the selections aremade? Express your ratio as a fraction or with a colon.

Answers

According to the information given in the exercise:

- Group A contains a total of 155 people.

- One-fifth of that people will be selected to win $20 fuel cards.

- The total number of people in Group B is 686.

Then, you can determine that the number of people that will be selected to win $20 fuel cards is:

[tex]winners_A=\frac{1}{5}(155)=31[/tex]

Therefore, the number of nonwinners in Group A is:

[tex]N.winners_A=155-31=124[/tex]

You know that Group B will receive the same number of fuel cards. Therefore, its number of nonwinners is:

[tex]N.winners_B=686-31=655[/tex]

Knowing all this information, you can set up the following ratio of nonwinners in Group A to nonwinners in Group B after the selections are made:

[tex]\frac{124}{655}[/tex]

Hence, the answer is:

[tex]\frac{124}{655}[/tex]


3. Convert the angle 3π/4 to degrees.

Answers

Answer:

135°

Step-by-step explanation:

To convert an angle from radians to degrees, multiply by [tex]180/\pi[/tex].

[tex]\frac{3\pi}{4} \cdot \frac{180}{\pi}=135^{\circ}[/tex]

Find the value of x assume the triangles are the same

Answers

[tex]x=44[/tex]

1) In this problem, we need to find the constant of proportionality assuming these triangles are similar. So let's divide each corresponding leg:

[tex]\frac{22}{18}=\frac{33}{27}\Rightarrow\:k=\frac{11}{9}[/tex]

2) So, based on that constant of proportionality (k) we can find the missing leg.

[tex]\begin{gathered} x\div\frac{11}{9}=36 \\ \\ x\cdot\frac{9}{11}=36 \\ \\ 11\times\frac{9}{11}x=36\times11 \\ \\ 9x=396 \\ \\ \frac{9x}{9}=\frac{396}{9} \\ \\ x=44 \end{gathered}[/tex]

Note that since the triangle on the top is larger than the one on the bottom, we can tell that x must be larger than 36.

Other Questions
Help?(10 points for answer) the difficulty voters face in evaluating whether candidates for office share their values and preferences, and whether candidates are corrupt, creates a/an . vandelay industries employs chemical and manufacturing experts who are among the best in the world at developing and producing products that contain latex. vandelay industries is highly efficient and outperforms its competitors which results in a(n) over its competition. Liam's monthly bank statement showed the following deposits and withdrawals.If Liam's balance in the account was $62.45 at the beginning of the month, what was the account balance at the end of the month? find the coordinates of the vertex of the following parabola algebraically. write your answer as an (x,y) point..y=x+9 Jocelyn graphs a linear function that passes through three distinct points: A, B, and C. The coordinates ofpoint A (-3, -3) and point C (3,5) are shown.What are the possible coordinates of point B for Jocelyns linear function? Last year, Trey opened an investment account with $8800. At the end of the year, the amount in the account had decreased by 6.5%. How much is this decrease in dollars? How much money was in his account at the end of last year?Decrease in amount:$Year-end amount:$ what does the lack of sexual dimorphism in gibbons indicate? group of answer choices the polygamous social structure in gibbons unequal access to resources within their environment decreased competition for mates in a monogamous social structure the fact that sexual dimorphism is not related to social structure in gibbons The coordinate pairs for triangle ABC are A(1,2), B(4,5), C(2,2). It undergoes a translation of 2 units right and 1 unit 1 up. Write down the coordinates of A' examples of social enterprise obligations to the environment include all of the following except a. pollution control. b. restoration of the environment. c. recycling efforts. small firms generally have relatively high loadings (high betas) on the smb (small minus big) factor. a. explain why this is not surprising. b. now suppose two unrelated small firms merge. each will be operated as an independent unit of the merged company. would you expect the stock market behavior of the merged firm to differ from that of a portfolio of the two previously independent firms? c. how does the merger affect market capitalization? d. what is the prediction of the fama-french model for the risk premium on the merged firm compared to the weighted average of the two component companies? e. do we see here a problem in applying the ff model? In order to throw the voice into the audience so that it penetrates to the utmost reaches of the theatre, the actor must?. List the three systems of government and define them a rope passing through a capstan on a dock is attached to a boat offshore. the rope is pulled in at a constant rate of 6 ft/s and the capstan is 5 ft vertically above the water. how fast is the boat traveling when it is 12 ft from the dock? Which of these word pairs contain a closed compound word and a hyphenated compound word that are written correctly? a.swimming-pool, overturned b.half-hearted, ice cream c.post office, runners-up d.watermelon, brother-in-law What do you diagram to analyze orbital motion ? a triangular lot is 130 ft on one side and has a property line of length 700 ft. Find the area of the lot in acres. Two drops of mercury each has a charge on 5.44 nC and a voltage of 235.15 V. If the two drops are merged into one drop, what is the voltage on this drop? Analyze the origins and development of slavery in Britain's North American colonies in the period 1607 to 1776. according to multiple studies, including those by the american association of university women and the pew research center, on average, women are paid approximately which percent of what men are paid? select one: a. 80 b. 100 c. 90 d. 70